summaryrefslogtreecommitdiff
path: root/exercise02
diff options
context:
space:
mode:
Diffstat (limited to 'exercise02')
-rw-r--r--exercise02/exercise02.tex299
1 files changed, 295 insertions, 4 deletions
diff --git a/exercise02/exercise02.tex b/exercise02/exercise02.tex
index dcd06d2..51099f7 100644
--- a/exercise02/exercise02.tex
+++ b/exercise02/exercise02.tex
@@ -3,7 +3,7 @@
\section*{Exercise 2}
\begin{question}[subtitle={Mono-chromatic Signals}]
- Given is a mono-chromatic signal $u(t)$:
+ A mono-chromatic signal $u(t)$ is given:
\begin{equation*}
u(t) = \SI{2}{V} \cdot \cos\left(2 \pi \cdot \SI{1}{MHz} \cdot t + \frac{\pi}{2} \right)
\end{equation*}
@@ -37,17 +37,103 @@
\end{tasks}
\end{solution}
-\begin{question}[subtitle={Using the Fourier transform}]
+\begin{question}
+ The following periodic signal is given.
+ \begin{figure}[H]
+ \centering
+ \begin{tikzpicture}
+ \begin{axis}[
+ height={0.25\textheight},
+ width=0.8\linewidth,
+ scale only axis,
+ xlabel={$t \text{ in } \si{s}$},
+ ylabel={$x(t) \text{ in } \si{V}$},
+ %grid style={line width=.6pt, color=lightgray},
+ %grid=both,
+ grid=none,
+ legend pos=north east,
+ axis y line=middle,
+ axis x line=middle,
+ every axis x label/.style={
+ at={(ticklabel* cs:1.05)},
+ anchor=north,
+ },
+ every axis y label/.style={
+ at={(ticklabel* cs:1.05)},
+ anchor=east,
+ },
+ xmin=-3.5,
+ xmax=3.5,
+ ymin=-0.5,
+ ymax=1.1,
+ xtick={-3.0, -2.5, ..., 3.0},
+ %ytick={0, 0.5, ..., 1.5},
+ %xticklabels={0, 1, $t_0$, 3, 4, ..., 10}
+ ]
+ \addplot[blue, thick] coordinates {(-3.0,-0.2) (-2.5,-0.2)};
+ \addplot[blue, thick] coordinates {(-2.5,0.8) (-1.5,0.8)};
+ \addplot[blue, thick] coordinates {(-1.5,-0.2) (-0.5,-0.2)};
+ \addplot[blue, thick] coordinates {(-0.5,0.8) (0.5,0.8)};
+ \addplot[blue, thick] coordinates {(0.5,-0.2) (1.5,-0.2)};
+ \addplot[blue, thick] coordinates {(1.5,0.8) (2.5,0.8)};
+ \addplot[blue, thick] coordinates {(2.5,-0.2) (3.0,-0.2)};
+
+ \addplot[blue, thick, dashed] coordinates {(-2.5,-0.2) (-2.5,0.8)};
+ \addplot[blue, thick, dashed] coordinates {(-1.5,0.8) (-1.5,-0.2)};
+ \addplot[blue, thick, dashed] coordinates {(-0.5,-0.2) (-0.5,0.8)};
+ \addplot[blue, thick, dashed] coordinates {(0.5,0.8) (0.5,-0.2)};
+ \addplot[blue, thick, dashed] coordinates {(1.5,-0.2) (1.5,0.8)};
+ \addplot[blue, thick, dashed] coordinates {(2.5,0.8) (2.5,-0.2)};
+ \end{axis}
+ \end{tikzpicture}
+ \end{figure}
+
+ \begin{tasks}
+ \task
+ Find a functional expression for the above signal!
+ \task
+ What is the base frequency?
+ \task
+ Find the real-valued Fourier series coefficients $a_n$ and $b_m$!
+ \task
+ Find the complex-valued Fourier series coefficients $\underline{c}_n$!
+ \task
+ Plot the amplitude and phase spectra for $-5 \leq n \leq 5$!
+ \end{tasks}
+\end{question}
+
+\begin{solution}
+ \begin{tasks}
+ \task
+ \begin{equation*}
+ x(t) = \begin{cases}
+ \SI{-0.2}{V}, &\quad \text{ if} \; \left(\SI{-0.5}{s} + n \cdot \SI{2}{s}\right) \leq t < \left(\SI{0.5}{s} + n \cdot \SI{2}{s}\right) \\
+ \SI{-0.2}{V}, &\quad \text{ if} \; \left(\SI{0.5}{s} + n \cdot \SI{2}{s}\right) \leq t < \left(\SI{1.5}{s} + n \cdot \SI{2}{s}\right) \\
+ \end{cases} \qquad \forall \; n \in \mathbb{Z}
+ \end{equation*}
+
+ \task
+ \begin{itemize}
+ \item Period: $T_0 = \SI{2}{s}$
+ \item Base frequency: $f_0 = \SI{0,5}{Hz}$
+ \item Base angular frequency: $\omega_0 = \SI{3.14}{s^{-1}}$
+ \end{itemize}
+
+ %TODO
+ \end{tasks}
+\end{solution}
+
+\begin{question}[subtitle={Using the Fourier Transform}]
Derive the Fourier transform, without using the duality, of
\begin{tasks}
\task
- Derive the Fourier transform of the time shift, without using the duality!
+ the time shift
\begin{equation*}
\mathcal{F}\left\{\underline{f}(t - t_0)\right\}
\end{equation*}
\task
- Derive the Fourier transform of the frequency shift, without using the duality!
+ the frequency shift
\begin{equation*}
\mathcal{F}\left\{e^{j \omega_0 t} \underline{f}(t)\right\}
\end{equation*}
@@ -119,3 +205,208 @@
% We obtain the same result as in b). The duality works. \acs{QED}
\end{tasks}
\end{solution}
+
+\begin{question}[subtitle={Fourier Transform of Signals}]
+ The following signal is given!
+ \begin{equation*}
+ \underline{x}(t) = j \cos\left(\omega_0 t\right) - \sin\left(\omega_0 t\right)
+ \end{equation*}
+
+ \begin{tasks}
+ \task
+ What is the Fourier transform of the signal?
+ \task
+ Plot the amplitude and phase spectra!
+ \task
+ Why does the spectrum not fulfil the symmetry rules?
+ \end{tasks}
+\end{question}
+
+\begin{solution}
+ \begin{tasks}
+ \task
+ \begin{equation*}
+ \underline{X}\left(j \omega\right) = j 2 \pi \delta\left(\omega - \omega_0\right)
+ \end{equation*}
+
+ \task
+ \begin{minipage}{0.45\linewidth}
+ \begin{figure}[H]
+ \centering
+ \begin{tikzpicture}
+ \begin{axis}[
+ height={0.25\textheight},
+ width=0.9\linewidth,
+ scale only axis,
+ xlabel={$\omega \text{ in } \si{Hz}$},
+ ylabel={$\left|\underline{X}\left(j \omega\right)\right| \text{ in } \si{V/Hz}$},
+ %grid style={line width=.6pt, color=lightgray},
+ %grid=both,
+ grid=none,
+ legend pos=north east,
+ axis y line=middle,
+ axis x line=middle,
+ every axis x label/.style={
+ at={(ticklabel* cs:1.05)},
+ anchor=north,
+ },
+ every axis y label/.style={
+ at={(ticklabel* cs:1.05)},
+ anchor=east,
+ },
+ xmin=-2.5,
+ xmax=2.5,
+ ymin=0,
+ ymax=6.5,
+ xtick={-1, 0, 1},
+ xticklabels={$-\omega_0$, 0, $\omega_0$},
+ ytick={0, 6.28},
+ yticklabels={0, 6.28},
+ ]
+ \addplot[blue, thick] coordinates {(1,0) (1,6.28)};
+ \addplot[blue, thick, only marks, mark=o] coordinates {(1,6.28)};
+ \end{axis}
+ \end{tikzpicture}
+ \end{figure}
+ \end{minipage}
+ \hfill
+ \begin{minipage}{0.45\linewidth}
+ \begin{figure}[H]
+ \centering
+ \begin{tikzpicture}
+ \begin{axis}[
+ height={0.25\textheight},
+ width=0.9\linewidth,
+ scale only axis,
+ xlabel={$\omega \text{ in } \si{Hz}$},
+ ylabel={$\arg\left(\underline{X}\left(j \omega\right)\right) \text{ in } \si{\degree}$},
+ %grid style={line width=.6pt, color=lightgray},
+ %grid=both,
+ grid=none,
+ legend pos=north east,
+ axis y line=middle,
+ axis x line=middle,
+ every axis x label/.style={
+ at={(ticklabel* cs:1.05)},
+ anchor=north,
+ },
+ every axis y label/.style={
+ at={(ticklabel* cs:1.05)},
+ anchor=east,
+ },
+ xmin=-2.5,
+ xmax=2.5,
+ ymin=-3.5,
+ ymax=3.5,
+ xtick={-1, 0, 1},
+ xticklabels={$-\omega_0$, 0, $\omega_0$},
+ ytick={-3.14, -1.57, 0, 1.57, 3.14},
+ yticklabels={$-\pi$, $-\frac{\pi}{2}$, 0, $\frac{\pi}{2}$, $\pi$},
+ ]
+ \addplot[blue, thick] coordinates {(1,0) (1,1.57)};
+ \addplot[blue, thick, only marks, mark=o] coordinates {(1,1.57)};
+ \end{axis}
+ \end{tikzpicture}
+ \end{figure}
+ \end{minipage}
+
+ \task
+ The signal is complex-valued. The symmetry rules only apply for real-valued signals.
+ \end{tasks}
+\end{solution}
+
+\begin{question}
+ The following circuit is given.
+ \begin{figure}[H]
+ \centering
+ \begin{circuitikz}
+ \draw (0, 0) to[L, l=$L$, o-] ++(2,0) to[short, *-o] ++(2,0);
+ \draw (2, 0) to[C, l=$C$, -*] ++(0,-2);
+ \draw (0, -2) to[short, o-o] ++(4,0);
+
+ \draw (0, 0) to[open, v=$u_i(t)$] (0, -2);
+ \draw (4, 0) to[open, v^=$u_o(t)$] (4, -2);
+ \end{circuitikz}
+ \end{figure}
+ \begin{tasks}
+ \task
+ Find a differential equation which connects $u_i(t)$ and $u_o(t)$!
+ \task
+ Determine the transfer function $\underline{H} \left(j \omega\right)$!
+ \task
+ Calculate the impulse response!
+ \task
+ Is the system causal? Why?
+ \task
+ What filter characteristic does the circuit have? Which order does the system have?
+ \end{tasks}
+\end{question}
+
+\begin{question}
+ \begin{figure}[H]
+ \centering
+ \begin{circuitikz}
+ \draw (0, 0) to[C, l=$C$, o-] ++(2,0) to[short, *-o] ++(2,0);
+ \draw (2, 0) to[R, l=$R$, -*] ++(0,-2);
+ \draw (0, -2) to[short, o-o] ++(4,0);
+
+ \draw (0, 0) to[open, v=$u_i(t)$] (0, -2);
+ \draw (4, 0) to[open, v^=$u_o(t)$] (4, -2);
+ \end{circuitikz}
+ \end{figure}
+
+ The high-pass filter has the following transfer function:
+ \begin{equation}
+ \underline{H}\left(j \omega\right) = \frac{j \omega RC}{j \omega RC + 1}
+ \end{equation}
+ with
+ \begin{itemize}
+ \item $R = \SI{100}{\ohm}$
+ \item $C = \SI{470}{nF}$
+ \end{itemize}
+
+ \begin{tasks}
+ \task
+ Which order does the system have?
+ \task
+ What are the poles and zeroes of the system? Is the system stable?
+ \task
+ Determine and plot the amplitude response $\left|\underline{H}\left(j \omega\right)\right|$!
+ \task
+ Determine and plot the phase response $\arg\left(\underline{H}\left(j \omega\right)\right)$!
+ \task
+ The following signal is applied to the input of the system $u_i(t)$.
+ \begin{equation}
+ u_i(t) = \SI{2}{V} \cos\left(2 \pi \cdot \SI{25}{kHz} \cdot t\right)
+ \end{equation}
+ Calculate the output signal $u_o(t)$ as either a time domain function or a phasor!
+ \end{tasks}
+\end{question}
+
+\begin{solution}
+ \begin{tasks}
+ \task
+ 1st order
+ \begin{itemize}
+ \item Only one capacity as a memorizing component
+ \item Highest exponent is $1$.
+ \end{itemize}
+
+ \task
+ Replace $j \omega$ by $\underline{s}$.
+ \begin{itemize}
+ \item Zero: $\underline{s}_0 = 0$ (Numerator of $\underline{H}\left(j \omega\right)$ must be zero)
+ \item Pole: Denominator of $\underline{H}\left(j \omega\right)$ must be zero
+ \begin{equation*}
+ \begin{split}
+ \underline{s}_{\infty} &= -\frac{1}{RC} \\
+ &= -\frac{1}{\SI{100}{\ohm} \cdot \SI{470}{nF}} \\
+ &= \SI{-21276.6}{s^{-1}}
+ \end{split}
+ \end{equation*}
+ \end{itemize}
+ The system is stable because the real part of its pole is negative.
+
+ %TODO
+ \end{tasks}
+\end{solution}